Hi Brilliant! I've decided to start the first ever summation contest here.
The aims of the Summation contest are to improve skills in the computation of sums, to learn from each other as much as possible, and of course to have fun. Anyone here may participate in this contest.
The rules are as follows:
I will start by posting the first problem. If there is a user solves it, then they must post a new one.
You may only post a solution of the problem below the thread of problem and post your proposed problem in a new thread. Put them separately.
Only make substantial comment that will contribute to the discussion.
Make sure you know how to solve your own problem before posting it in case there is no one can answer it within 48 hours, then you must post the solution and you have a right to post another problem.
If the one who solves the last problem does not post his/her own problem after solving it within a day, then the one who has a right to post a problem is the last solver before him/her.
You are NOT allowed to post a multiple summation problem.
Problems must be purely of summation. They shouldn't have integrals and products in them. Solutions can follow methods that use integrals and products.
It is NOT compulsory to post original problems. But make sure it has not been posted on brilliant.
There is no restriction in the standard of summations.
Please post your solution and your proposed problem in a single new thread.
Format your post is as follows:
1 2 3 4 5 6 7 |
|
The comments will be easiest to follow if you sort by "Newest":
See Part-2.
Easy Math Editor
This discussion board is a place to discuss our Daily Challenges and the math and science related to those challenges. Explanations are more than just a solution — they should explain the steps and thinking strategies that you used to obtain the solution. Comments should further the discussion of math and science.
When posting on Brilliant:
*italics*
or_italics_
**bold**
or__bold__
paragraph 1
paragraph 2
[example link](https://brilliant.org)
> This is a quote
\(
...\)
or\[
...\]
to ensure proper formatting.2 \times 3
2^{34}
a_{i-1}
\frac{2}{3}
\sqrt{2}
\sum_{i=1}^3
\sin \theta
\boxed{123}
Comments
Problem 2: Prove that n≥1∑n2(−1)n(γ+ψ(3n+21))=263ζ(3)+ζ(2)log2−14πG
G is Catalan's constant.
This problem has been solved by Ishan Singh.
Proposition:
Log in to reply
Solution Of Problem 2 :
Proof : It is easy to see that
n=1∑∞nHnxn=Li2(x)+21ln2(1−x)
Dividing by x and integrating, we have,
n=1∑∞n2Hnxn=∫xLi2(x)dx+21∫xln2(1−x)dx
=Li3(x)+21[ln2(1−x)lnx]+∫1−xlnxln(1−x)dx
Let x=1−t
⟹n=1∑∞n2Hnxn=Li3(x)+21[ln2(1−x)lnx]−∫tln(1−t)lntdt
Now,
∫tln(1−t)lntdt=lntLi2(t)−∫tLi2(t)dt
=lntLi2(t)−Li3(t)
Substituting back, we have,
n=1∑∞n2Hnxn=Li3(x)−Li3(1−x)+ln(1−x)Li2(1−x)+21lnxln2(1−x)+C
Putting x=0, we get C=ζ(3), thus,
n=1∑∞n2Hnxn=Li3(x)−Li3(1−x)+ln(1−x)Li2(1−x)+21lnxln2(1−x)+ζ(3)□
Now,
S=n=1∑∞n2(−1)n(γ+ψ(3n+21))=−2ln2n=1∑∞n2(−1)n+n=1∑∞k=1∑3n2k−12
Also,
k=1∑3n2k−11=k=1∑6nk1−k=1∑3n2k1
k=1∑3n2k−11=H6n−21H3n
⟹S=ζ(2)ln2+n=1∑∞n2(−1)n[2H6n−H3n]
Using cube roots and sixth roots of unity on the proposition, we have,
n=1∑∞n2(−1)n[2H6n−H3n]=263ζ(3)−14πG
⟹S=ζ(2)ln2+263ζ(3)−14πG
PROBLEM 11
Evaluate n=1∑∞(−1)n2n+1Hn.
This problem has been solved by Jake Lai and Aditya Kumar.
Log in to reply
It is well-known that −1−xln(1−x)=n=1∑∞Hnxn. So
n=1∑∞2n+1(−1)nHn=∫01n=1∑∞Hn(−x2)n dx=∫011+x2−ln(1+x2) dx=∫0π/4−ln(1+tan2u) du=∫0π/4−ln(sec2u) du=∫0π/42lncosu du=G−2πln2(Substitute u=tan−1x)
where G is Catalan's constant.
Log in to reply
Reversing the order of integration and summation to obtain the second line of your argument takes a bit of justification. The generating function series is not monotonic, nor is it uniformly bounded by an integrable function, so the result is not automatic.
You need to integrate from 0 to x<1, obtaining N=1∑∞2n+1(−1)nHnx2n+1=2∫0tan−1xlncosudu and then let x tend to 1−, using Abel's Theorem to get the answer (having used the Alternating Series Test to prove that the series ∑n(−1)n2n+1Hn is convergent).
Anyway, you are up for the next question!
Alternate Method:
n=1∑∞2n+1(−1)nHn=n=1∑∞4n+12H2n−n=1∑∞2n+1Hn
Then we can use appropriate generating functions for evaluating it. The integrals have to be carefully handled in order to get the solution.
SOLUTION OF PROBLEM 1 :
n≥0∑(−1)nψ(3)(2n+2)======3!n≥0∑(−1)nk≥0∑(k+2n+2)416n,k≥0∑(k+2n+2)4(−1)n=6K≥0∑⎝⎛n=0∑⌊K/2⌋(−1)n⎠⎞(K+2)416K≥0,K≡0,1(4)∑(K+2)4183K≥0∑(2K+1)41+6K≥0∑(4K+3)4183(1−161)901π4+1283ζ(4,23)2561π4+2561ψ(3)(43)
Special Problem :
Evaluate
r=0∑n(rn)(−1)r
Log in to reply
Solution to Special Problem : Method 1
S=r=0∑n(rn)(−1)r
=r=0∑n(−1)rn!r!(n−r)!
=r=0∑n(−1)rn!r!(n−r)!(n−r+1+r+1)×(n+2)1
=n!(n+2)1r=0∑n[(−1)rr!(n−r+1)!+(−1)r(r+1)!(n−r)!]
=n!(n+2)1r=0∑n[(−1)rr!(n−r+1)!−(−1)r+1(r+1)!(n−r)!]
=n!(n+2)1r=0∑n(Tr−Tr+1)(*)
where Tr=(−1)rr!(n−r+1)!
Clearly, (∗) is a telescoping series. Evaluating it, we have,
S=(−1)nTn+1+T0
=(−1)nn!(n+2)(n+1)!+n!(n+2)(n+1)!
Since n is even,
∴S=2n+2n+1
Method 2
Consider the following Lemma.
Proof : S=r=0∑k(rk)(−1)r
=r=0∑k(k−rk)(−1)k−r
(∵r=0∑nf(r)=r=0∑nf(n−r))
=−r=0∑k(rk)(−1)r
⟹S=−S
⟹S=0
This completes the proof of the Lemma.
Now, since n is even, n+1 is odd.
Using the Lemma, we have,
r=0∑n+1(rn+1)(−1)r=0(1)
Let,
J=r=0∑n(rn)(−1)r(2)
Operating (1)+(2),
⟹J+0=r=0∑n(rn)(−1)r+r=0∑n+1(rn+1)(−1)r
=r=0∑n(rn)(−1)r+r=0∑n(rn+1)(−1)r+(n+1n+1)(−1)
=r=0∑n(rn)(−1)r+r=0∑n(n−rn+1)(−1)n−r−1(∵r=0∑nf(r)=r=0∑nf(n−r))
=r=0∑n(rn)(−1)r+r=0∑n(r+1n+1)(−1)r−1
=r=0∑n(−1)r⎩⎪⎪⎨⎪⎪⎧(rn)1+n+1r+1⋅(rn)1⎭⎪⎪⎬⎪⎪⎫−1[∵(rn)=rn(r−1n−1) & (rn)=(n−rn)]
⟹J=(n+1n+2)r=0∑n(rn)(−1)r+n+11r=0∑n(rn)(−1)r⋅r−1
⟹J+1=(n+1n+2)J+n+11= G (let)r=0∑n(rn)(−1)r⋅r
⟹J+1=(n+1n+2)J+n+1G(3)
Now,
G=r=0∑n(rn)(−1)r⋅r
=r=0∑n(n−rn)(−1)n−r⋅(n−r)
=r=0∑n(rn)(−1)r⋅(n−r)
=nr=0∑n(rn)(−1)r−r=0∑n(rn)(−1)r⋅r
⟹G=n⋅J−G
⟹G=2Jn(4)
From (3) & (4),
⟹J+1=(n+1n+2)J+2(n+1)Jn
∴J=2n+2n+1□
Start by writing (rn)−1=(n+1)∫01ur(1−u)n−rdu.
Multiply both sides by (−1)r and adding over r gives:
r=0∑n(rn)(−1)r===(n+1)∫01r=0∑n(−u)r(1−u)n−rdu(n+1)∫01[(1−u)n+1+(−1)nun+1]dun+2n+1(1+(−1)n)=⎩⎨⎧0,n odd2n+4n+1,n even
Log in to reply
This is the same method which Ishan talked about. Thanks for posting!
Problem 3:
Evaluate :
r=0∑n[(rn)⋅sinrx⋅cos(n−r)x]
This problem has been solved by Deeparaj Bhat and Aditya Kumar.
Log in to reply
Call the given sum S.
Then, by using the definition of trigonometric functions via complex exponentials, we have S=4i1r=0∑n(rn)(einx−e−inx+e(2r−n)ix−e−(2r−n)ix)
Now, using binomial theorem and a bit of simplification, we get that S=2n−1sin(nx)
PS: I don't have good summation problems. So, anyone can post in my behalf. :)
Log in to reply
Alternate Solution:
We use the identity: r=0∑nf(r)=r=0∑nf(n−r)
We take:
S=r=0∑n[(rn)⋅sinrx⋅cos(n−r)x](1)
On applying the identity, we get
S=r=0∑n[(n−rn)⋅sin(n−rx)⋅cosrx](2)
On adding equations 1 and 2, we get:
2S=r=0∑n(rn)sinnx
Therefore, S=2n−1sinnx
Problem 6 :
Prove That
r=1∑nr(−1)r−1(rn)=Hn
Notation : Hn denotes the Harmonic Number.
This problem has been solved by Aditya Kumar.
Log in to reply
Consider the binomial expansion:
r=0∑n(−x)r(rn)=(1−x)n
We'll write it as:
r=1∑n(−x)r(rn)=(1−x)n−1
On dividing both the sides by −x, we get:
r=1∑n(−x)r−1(rn)=x1−(1−x)n
On integrating both the sides w.r.t. x from 0 to 1, we get:
r=1∑nr(−1)r−1(rn)=∫01x1−(1−x)ndx=∫011−x1−xndx=Hn
Solution To Problem 6 :
S=r=1∑nr(−1)r−1(rn)
Note that, k=r∑n(r−1k−1)=(rn), it follows,
S=r=1∑nr(−1)r−1k=r∑n(r−1k−1)
Since (r−1k−1)=kr(rk), we have,
S=r=1∑nk=r∑nk(−1)r−1(rk)
Interchanging order of summation, we have,
S=k=1∑nr=1∑kk(−1)r−1(rk)
=k=1∑nk1
=Hn□
Problem 12:
Prove the following for ∣x∣<274.
n=0∑∞(n3n)xn=4−27x2cos(31sin−1(233x))
This problem has been solved by Mark Hennings and Ishan Singh.
Log in to reply
Solution to Problem 12:
By the multiplication formula for the Gamma function, (3n)!=Γ(3n+1)=2π132n+21Γ(n+31)Γ(n+32)Γ(n+1) and hence (n3n)=2π33n+21×Γ(2n+1)Γ(n+31)Γ(n+32)=2π32n+21B(n+32,n+31) Thus S=n=0∑∞(n3n)xn==2π3n=0∑∞32nxn∫01un−31(1−u)n−32du2π3∫011−27xu(1−u)u−31(1−u)−32du=2π3∫011−4sin23αu(1−u)u−31(1−u)−32du where 0<α<61π is such that sin3α=2127x. The substitutions u=sin2θ and then x=tanθ yield S===2π3∫021π1−sin23αsin22θsin−32θcos−34θ×2sinθcosθdθ=π3∫021π1−sin23αsin22θtan31θdθπ3∫0∞(1+x2)2−4sin23αx2x31(1+x2)dx=π3∫0∞(x2+2xsin3α+1)(x2−2xsin3α+1)x31(1+x2)dxπ3∫0∞x31f(x)dx=π3I where the meromorphic function f(z)=(z2+2zsin3α+1)(z2−2zsin3α+1)z2+1 has simple poles at each of the points in R={±ie3iα,±ie−3iα}.
Cutting the complex plane along the positive real axis, and integrating around the "keyhole contour" γ1+γ2−γ3−γ4 where
for any 0<ϵ<1<R, then (∫γ1+∫γ2−∫γ3−∫γ4)z31f(zdz=2πiu∈R∑Resz=uz31f(z) so that (1−ω)∫ϵRx31f(x)dx+(∫γ2−∫γ4)z31f(z)dz=2πiu∈R∑Resz=uz31f(z) Letting ϵ→0 and R→∞, we obtain (1−ω)I=2πiu∈R∑Resz=uz31f(z) and hence I=3(2cos2α−1)π=3(4cos2α−3)π=3cos3απcosα so that S=cos3αcosα=4−27x2cos(31sin−1(233x)) as required.
Solution To Problem 12 :
Proof : Note that,
n=0∑∞Un(a)(−x)n=x2+2ax+11
where Un(x) is the Chebyshev Polynomial of the second kind.
⟹n=0∑∞Un(a)Γ(n+1)n!(−x)n=x2+2ax+11
Using Ramanujan Master Theorem, we have,
f(a,z)=sinπzπU−s(a)
=sinπzπsin(cos−1(a))sin((1−z)cos−1(a))□
Now, using Gamma Triplication Formula,
S=n=0∑∞(n3n)xn=2π3∫0∞(x2+2ax+1)(x2−2ax+1)x31(1+x2)dx
where a=233x
Using Partial Fraction and the Proposition, we have,
S=2π3[21f(31,−a)+21f(37,−a)−4a1f(34,−a)−4a1f(310,−a)+21f(31,a)+21f(37,a)+4a1f(34,a)+4a1f(310,a)]
After simplification, we have,
S=4−27x2cos(31sin−1(233x))□
PROBLEM 13:
Show that n=1∑∞2n(Hn(2))2=3601π4−61π2ln22+61ln42+2Li4(21)+ζ(3)ln2
This problem has been solved by Aditya Kumar
Log in to reply
Solution to Problem 13:
Just to see how nasty the last integral in this problem can get, here are the associated indefinite integrals!
Start with B(x)===n=1∑∞(n+1)2Hn(2)xn+1=∫0x(∫0un=1∑∞Hn(2)vndv)udu∫0x(∫0u1−vLi2(v)dv)udu=∫0x(∫vxu(1−v)Li2(v)du)dv∫0xln(vx)1−vLi2(v)dv Now (differentiate back to check): ∫0x1−vLi2(v)dv∫0x1−vLi2(v)lnvdv==−31π2ln(1−x)+ln2(1−x)lnx+ln(1−x)Li2(x)+2Li3(1−x)−2ζ(3)451π4−61π2ln2(1−x)+41ln4(1−x)+21ln2(1−x)ln2x+(ln2(1−x)−ln(1−x)lnx+ln2x)Li2(x)−21Li2(x)2+ln2(1−xx)Li2(−1−xx)+2ln(1−x)Li3(1−x)−2lnxLi3(x)+2ln(1−x)Li3(−1−xx)−2lnxLi3(−1−xx)−2Li4(1−x)+2Li4(x)+2Li4(−1−xx) and hence B(21)=14401π4−241π2ln22+241ln42+41ζ(3)ln2 Now consider A(x)===n=1∑∞(Hn(2))2xn=n=1∑∞(Hn−1(2)+n21)2xnn=1∑∞n=1∑∞(Hn(2))2xn+1+2n=1∑∞(n+1)2Hn(2)xn+1+Li4(x)xA(x)+2B(x)+Li4(x) so that n=1∑∞2n(Hn(2))2==A(21)=4B(21)+2Li4(21)3601π4−61π2ln22+61ln42+2Li4(21)+ζ(3)ln2 as required.
Log in to reply
I have solved the indefinite integrals by using Landen's Identity.
I was wondering whether we can extend the result to other weights too?
The main integral to evaluate here is ∫xln2(1−x)lnxdx. Rest of the integrals follow from this using Euler's Reflection Formula and/or IBP. The integral ∫1−xLi2(x)dx is evaluated in my solution of Problem 2.
Can you please help me with this query which is related to the above integrals?
Solution to Problem 13:
Consider the function: f(x)=n=1∑∞(Hn(2))2xn.
f(x)=n=1∑∞(Hn−1(2)+n21)2xn
f(x)=n=1∑∞(Hn−1(2))2xn+Li4(x)+2n=1∑∞n2Hn−1(2)xn
On changing the summation index from n to n+1, we get:
f(x)=xf(x)+Li4(x)+2n=1∑∞(n+1)2Hn(2)xn+1(...1)
We have to find n=1∑∞(n+1)2Hn(2)xn+1
n=1∑∞(n+1)2Hn(2)xn+1=∫0x[n=1∑∞(n+1)Hn(2)xn]dx(...2)
Now we have to find n=1∑∞(n+1)Hn(2)xn
I'll use the identity: n=1∑∞nHn(2)xn=∫0xt(1−t)Li2(t)dt.
Now, we differentiate w.r.t. x and get:
n=1∑∞Hn(2)xn−1=x(1−x)Li2(x)
We multiply both sides by x:
n=1∑∞Hn(2)xn=(1−x)Li2(x)
Now, we integrate both sides w.r.t. x from 0 to x:
n=1∑∞n+1Hn(2)xn+1=2Li3(1−x)−2Li2(1−x)ln(1−x)−Li2(x)ln(1−x)−ln(x)(ln(1−x))2−2ζ(3)
On dividing both sides by x:
n=1∑∞n+1Hn(2)xn=x2Li3(1−x)−x2Li2(1−x)ln(1−x)−xLi2(x)ln(1−x)−xln(x)(ln(1−x))2−x2ζ(3)
Now plugging this in eqn 2, we get:
n=1∑∞(n+1)2Hn(2)xn+1=∫0x[x2Li3(1−x)−x2Li2(1−x)ln(1−x)−xLi2(x)ln(1−x)−xln(x)(ln(1−x))2−x2ζ(3)]dx
Now plugging this in eqn 1, we get:
f(x)=1−xLi4(x)+1−x2∫0x[x2Li3(1−x)−x2Li2(1−x)ln(1−x)−xLi2(x)ln(1−x)−xln(x)(ln(1−x))2−x2ζ(3)]dx
Now we take x=21 and compute the integral. Then we finally get:
n=1∑∞2n(Hn(2))2=3601π4−61π2ln22+61ln42+2Li4(21)+ζ(3)ln2
The final integral is left to the reader's exercise. I didn't post it as it would've made this solution too long.
Log in to reply
I'm impressed. The last integral is a brute.
Log in to reply
n=1∑∞(Hn(2))2xn . The solution gets too long though.
In fact we can even generalize the result by finding the indefinite integral of the last one and in turn the generating functionLog in to reply
Problem 1:
Prove that: n=0∑∞(−1)nψ3(2n+2)=256π4+256ψ(3)(43).
This problem has been first solved by Aman Rajput and second by Mark Hennings.
Solution to Problem 1:
Use the relation km+1ψ3(kz)=0≤n≤k−1∑ψ3(z+n/k)
Which reduces
n≥0∑(−1)nψ3(2n+2)=k≥1∑(−1)k−1161[ψ3(k)+ψ3(k+21)
=2563[ζ(4,3/4)−ζ(4,5/4)−256+3π4]
=512−ψ3(1/4)+512ψ3(3/4)+2569π4
and thus , we have 256π4+256ψ3(3/4)
Problem 8: Show that n=0∑∞n4+41=81(1+πcothπ)
This problem has been solved by Ishan Singh and Aditya Kumar.
Log in to reply
Solution To Problem 8 :
Proof : Let S=n=0∑∞n2+11
⟹S=2i1n=0∑∞(n−i1−n+i1)
Using Digamma Regularization, we have,
S=2i1[ψ(i)−ψ(−i)]
Simplifying using Digamma Reflection Formula, we have,
S=21(1+iπcot(iπ))
=21(1+πcoth(π))□
Now,
J=n=0∑∞n4+41
=n=0∑∞(n2+2n+2)(n2−2n+2)1
Using Partial Fraction,
J=41n=0∑∞(n2−2n+2)1+81n=0∑∞(n2+2n+2n+2−n2−2n+2n)
=81+41n=1∑∞(n−1)2+11+81n=0∑∞(n2+2n+2n+2−n2−2n+2n)
=81+41n=0∑∞n2+11+81n=0∑∞(Tn+2−Tn)
where Tn=n2−2n+2n
Note that the latter sum telescopes. Evaluating it, we have,
n=0∑∞(Tn+2−Tn)=−1
⟹J=41n=0∑∞n2+11
=81(1+πcoth(π))□
Solution to Problem 8:
For any positive integer N>2, let CN be the positively oriented square contour with corners at ±(N+21)±i(N+21). Let R be the set {1+i,1−i,−1−i,−1+i}. Then, since πcotπz is a meromorphic function, periodic of period 1, with a simple pole of residue 1 at each integer, we see that 2πi1∫CNz4+4πcotπzdz===n=−N∑NResz=nz4+4πcotπz+u∈R∑Resz=uz4+4πcotπz−41+2n=0∑Nn4+41+u∈R∑Resz=uz4+4πcotπz−41+2n=0∑Nn4+41−161πu∈R∑ucotπu Since cotπ(ϵ+ηi)=−icothπη for ϵ,η∈{1,−1}, it follows that 2πi1∫CNz4+4πcotπzdz=−41+2n=0∑Nn4+41−41πcothπ Since πcotπz is uniformly bounded on CN for all integers N, we deduce that N→∞lim2πi1∫CNz4+4πcotπzdz=0 which gives the result.
Claim: a=1∑∞a4+4b48b4=bπcoth(bπ)−1
Proof: Read Brian Chen's comment here.
The result follows.
S=n=0∑∞n4+41
On splitting it using partial fractions, we get:
S=4i1{n=0∑∞n2−2i1−n=0∑∞n2+2i1}
We use the identity: n=0∑∞a2+n21=2a21+aπcothaπ. See the proof here.
On simplifying (exercise to readers), we get: n=0∑∞n4+41=81(1+πcothπ)
Problem 4:
Evaluate \substack0≤r≤nr≡1(mod3)∑(rn)
This problem has been solved by Ishan Singh.
Log in to reply
Solution Of Problem 4 :
Let S=\substack0≤r≤nr≡1(mod3)∑(rn)
Since,
r=1∑n(rn)xr=(1+x)n
⟹r=1∑n(rn)ωr=(1+ω)n(1)
r=1∑n(rn)ω2r=(1+ω2)n(2)
r=1∑n(rn)=2n(3)
where ω=ei3π.
Operating ω2⋅(1)+ω⋅(2)+(3), we have,
S=31[ω2(1+ω)n+ω(1+ω2)n+2n]
Similarly, we get,
where ω represents one of the non real mth roots of unity.
Problem 5:
Prove That r=2∑∞FrFr−1(−1)r=ϕ−1
Notation : Fr denotes Fibonacci Number.
This problem has been solved my Mark Hennings.
Log in to reply
Solution to Problem 5:
Since Fn=51[ϕn−(−ϕ−1)n] we have FnFn−11==(ϕn−(−ϕ−1)n)(ϕn−1−(−ϕ−1)n−1)5=(ϕ2n−(−1)n)(ϕ2n−2−(−1)n−1)5ϕ2n−1ϕ+ϕ−15[ϕ2n−(−1)n1+ϕ2n−2−(−1)n−11] and hence (the series telescopes): r=2∑∞FnFn−1(−1)n===ϕ+ϕ−15n=2∑∞[ϕ2n−(−1)n(−1)n−ϕ2n−2−(−1)n−1(−1)n−1]ϕ+ϕ−15×ϕ2+11=(ϕ2+1)25ϕ=(ϕ+2)25ϕϕ+1ϕ=ϕ−1 as required. I am happy to let Ishan set another problem; I am very busy at present.
Problem 7:
Prove that:
n=1∑∞Hn(r)zn=1−zLir(z)
Here Hn(r) is generalized harmonic number and Lir(z) is polylogarithm.
This problem has been solved by Mark Hennings and Deeparaj Bhat.
Log in to reply
Solution to Problem 7:
(1−z)n=1∑∞Hn(r)zn===n=1∑∞Hn(r)zn−n=1∑∞Hn(r)zn+1z+n=2∑∞[Hn(r)−Hn−1(r)]zn=z+n=2∑∞nrznLir(z) for ∣z∣<1.
Consider the following summation: Q.E.D.(1−z)n=1∑∞Hn(r)zn=n=1∑∞(Hn(r)−Hn−1(r))zn(letH0(r)=0)=Lir(z)
Problem 10:
Evaluate: k=1∑∞2k+3ζ(2k+1)−1
This problem has been solved by Mark Hennings.
Log in to reply
Solution to Problem 10:
The sum is S=k≥1∑2k+3ζ(2k+1)−1=k≥1∑n≥2∑(2k+3)n2k+11=n≥2∑F(n1) where F(x)=k≥1∑2k+3x2k+1=2x21ln(1−x1+x)−31x−x−1∣x∣<1 Thus S=n≥2∑(21n2ln(n−1n+1)−3n1−n) After a shed-load of simplification, the partial sum SN==n=2∑N+1(21n2ln(n−1n+1)−3n1−n)21(N+1)2ln(N+2)+21N2ln(N+1)+2ln(Γ(N+1)NG(N+1))−21ln2+34−31HN+1−21(N+1)(N+2) where G is the Barnes G-function. With the known asymptotics lnG(N+1)lnΓ(N+1)∼∼121−lnA+21Nln(2π)+21(N2−61)lnN−43N2(N+1)ln(N+1)−(N+1)−21ln(N+1)+21ln(2π)+12(N+1)1 as N→∞, where A is the Glaisher constant, we deduce that ln(Γ(N+1)NG(N+1))∼−lnA+41N(N+4)+21(N2−61)lnN−21N(2N+1)ln(N+1) as N→∞ and hence we can deduce (after even more simplification) that S=N→∞limSN=1213−2lnA−21ln2−31γ
Log in to reply
Awesome solution!
Solution To Problem 10 :
From the generating function of Riemann Zeta, we have,
n=1∑∞ζ(n+1) xn=−γ−ψ(1−x)
⟹n=1∑∞(1+(−1)n)ζ(n+1) xn=−2γ−ψ(1−x)−ψ(1+x)
⟹n=1∑∞ζ(2n+1) x2n+1=−γx−xψ(x)−2πxcot(πx)−21(∗)
Now,
S=k=1∑∞2k+3ζ(2k+1)−1
=∫01x[k=1∑∞(x2k+1ζ(2k+1)−x2k+1)]dx
Using (∗), we have,
S=−∫01(γx2+x2ψ(x)+2x+2πx2cot(πx)+1−x2x4)dx
=1213−2logA−21log2−3γ□
Problem 14:
Prove that:
n=1∑∞2nn2ψ1(n)=144019π4−24π2log2(2)−24log4(2)−4ζ(3)log(2)
This problem has been solved by Mark Hennings.
Log in to reply
Solution to Problem 14:
We note that n=1∑∞2nn2ψ(1)(n)===21ζ(2)−n=1∑∞2n+1(n+1)21[Hn(2)−ζ(2)]ζ(2)n=1∑∞2nn21−n=1∑∞2n+1(n+1)2Hn(2)ζ(2)[121π2−21ln22]−n=1∑∞(n+1)22n+1Hn(2) Going back to the previous problem, we have A(x)(1−x)A(x)====n=1∑∞(Hn(2))2xn=n=1∑∞(Hn−1(2)+n21)2xnn=1∑∞(Hn(2))2xn+1+2n=1∑∞(n+1)2Hn(2)xn+1+Li4(x)xA(x)+2n=1∑∞(n+1)2Hn(2)xn+1+Li4(x)2n=1∑∞(n+1)2Hn(2)xn+1+Li4(x) and hence, putting x=21, 21n=1∑∞2n(Hn(2))2=2n=1∑∞(n+1)22n+1Hn(2)+Li4(21) and so n=1∑∞(n+1)22n+1Hn(2)=14401π4−241π2ln22+241ln42+41ζ(3)ln2 so that n=1∑∞2nn2ψ(1)(n)=144019π4−241π2ln22−241ln42−41ζ(3)ln2 Someone else can post the next question.
Log in to reply
This question was essentially the same as the previous question. @Aditya Kumar If you set the next question, please post a new one.
Log in to reply
Problem 15:
Prove that:
n=1∑∞(n2+1)21=4π(coth(π)+πcsch2(π)−π2)
Log in to reply
Solution to Problem 15:
Refer to my solution to Problem 8 for some of the technical details required about CN and πcotπz.
Integrating (z2+1)2πcotπz around the positively-oriented square contour CN with corners at ±(N+21)±i(N+21), we deduce that 2πi1∫CN(z2+1)2πcotπzdz==(n=−N∑NResz=n+Resz=i+Resz=−i)(z2+1)2πcotπzdz1+2n=1∑N(n2+1)21+(Resz=i+Resz=−i)(z2+1)2πcotπz and Resz=±i(z2+1)2πcotπz=dzd(z±i)2πcotπz∣∣∣z=±i=−41π2cosech2π−41πcothπ so that 2πi1∫CN(z2+1)2πcotπzdz=1+2n=1∑N(n2+1)21−21π2cosech2π−21πcothπ Since the integral around CN tends to 0 as N→∞, the result follows.
As before, I pass on setting the next problem.
Log in to reply
Alternate Method:
n=1∑∞(n2+1)21=41n=1∑∞{(−1+in)21−−1+in1+(1+in)21+1+in1}
Here, we can use polygamma function and use their reflection formulas to get to the final result.
Sir I can't think of a good problem and Ishan is down with fever. Can you post the next problem here?
The next problem would be posted here.
PROBLEM 16: If 0<θ<2π, evaluate n=1∑∞ncosnθ Don't forget to justify the convergence!
Log in to reply
Solution To Problem 16 :
Let an=n1 and bn=cosnθ. Note that both sequences satisfy the criterion of Dirchlet's Test. Therefore, the sum S=n=1∑∞ncosnθ converges.
Now,
S=n=1∑∞ncosnθ
=ℜ(n=1∑∞neinθ)
=−ℜ(ln(1−eiθ))
=−ln∣∣∣∣2sin(2θ)∣∣∣∣□
The contest has shifted to this note.
Problem 9 :
Evaluate
n=1∑∞2n(Hn)2
Notation : Hn denotes the Harmonic Number.
This problem has been solved by Aditya Kumar.
Log in to reply
Solution to Problem 9:
I'll use the generating function: (1−t)log2(1−t)+Li2(t)=n=1∑∞Hn2tn
Substitute: t=21.
Hence the final answer is: (ln2)2+6π2
Log in to reply
Proof For Generating Function :
f(x)=n=1∑∞Hn2xn
Since Hn=Hn−1+n1⟹Hn2=Hn−12+n21+2nHn−1 and we have,
f(x)=n=1∑∞Hn2xn=n=1∑∞(Hn−12xn+n2xn+2nHn−1xn)
Changing summation index n−1↦n in the first and third sum, we have,
f(x)=xn=1∑∞Hnxn+Li2(x)+2n=1∑∞n+1Hnxn+1
⟹f(x)=xf(x)+Li2(x)+2n=1∑∞n+1Hnxn+1
Also,
n=1∑∞n+1Hnxn+1=∫0xn=1∑∞Hntndt
=−∫0x1−tln(1−t)dt
=2ln2(1−x)
⟹f(x)=xf(x)+Li2(x)+ln2(1−x)
⟹f(x)=1−xln2(1−x)+Li2(x)